LSAT and Law School Admissions Forum

Get expert LSAT preparation and law school admissions advice from PowerScore Test Preparation.

 Administrator
PowerScore Staff
  • PowerScore Staff
  • Posts: 8917
  • Joined: Feb 02, 2011
|
#31360
Complete Question Explanation

Our author opens with his conclusion that unemployment will soon decrease. He then goes on to tell us that significantly increased government spending will lead to this result, and that significantly decreased government spending will also lead to that same result. These are his premises for the claim in the first sentence.

At first this Justify the Conclusion question comes off like a False Dilemma flaw question, and test takers may have braced themselves for that question stem and answer choice as soon as they noticed that the author made the classic mistake of ignoring the possibility that things could stay the same (or at least not change significantly). The government might spend something close to what they have been spending, neither increasing nor decreasing significantly.

In order to justify the conclusion, we must make this dilemma real - we have to eliminate the possibility of spending staying close to current amounts. If spending cannot stay the same or somewhat close to it, then it must go up or down significantly, either of which would lead to the conclusion. That must be our prephrase - spending cannot stay as it is but must change significantly one way or the other.

Answer A: This is the correct answer. A perfect match for our prephrase, eliminating the dilemma and proving that one way or another, unemployment will decrease.

Answer B: There is no need to assume anything about what policies are currently being implemented or contemplated or rejected. All we need to assume is that spending will change significantly in one direction or another.

Answer C: While this answer looks reasonable and makes perfect sense on its own, it doesn't tell us with any certainty that there will be such increased demand for workers, and so this does not prove that unemployment will decrease soon. It only shows us one way in which that might come about.

Answer D: This answer is outside the scope of the argument and does nothing to help justify our conclusion about what will happen soon. It gives additional information about what could happen later, but that's not what we are looking for.

Answer E: This answer looks like a Mistaken Negation of the two conditional claims in the argument, and as such it cannot justify our conclusion.
 EmiliaGrace
  • Posts: 5
  • Joined: Jun 01, 2018
|
#46314
Hi, I honestly had trouble finding the conclusion when I took this test under timed conditions. I had originally chosen C has my answer choice. Could the premise be diagrammed using conditional reasoning?
 Alex Bodaken
PowerScore Staff
  • PowerScore Staff
  • Posts: 136
  • Joined: Feb 21, 2018
|
#46351
EmiliaGrace,

Thanks for the question! Sure, we could diagram this one out. Here's what it would look like:

Premise 1: Government spending signifcantly increases :arrow: unemployment decreases
Premise 2: Government spending significantly decreases :arrow: unemployment decreases
Conclusion: Unemployment will soon decrease.

Note that the conclusion is the first sentence of the stimulus here - I know you said in your question you had a tough time picking it out.

Obviously this is a bit of a strange argument: the economist is basically saying that whether government spending significantly increases or significantly decreases, unemployment will decrease. So in order to prove the conclusion, we are looking for an assumption that one of the two premises will occur. And that is what answer choice (A) gives us: it says that either premise 1 or premise 2 will occur, and we know that if either of those occur, then unemployment will decrease, thereby validating the economist's conclusion.

Hope that's helpful!
Alex
 altheaD
  • Posts: 17
  • Joined: Jun 08, 2018
|
#60079
Hi,
I get why A is a right answer, but I still can't understand why C is wrong. I mean, A is not the right answer so long as C isn't true either. And I thought C helps prove that unemployment will decrease,because, unless C assumption is true, the A option won't work either. So if A is contingent on C, then C is the necessary assumption...no?? help!! thank you!!
 Malila Robinson
PowerScore Staff
  • PowerScore Staff
  • Posts: 296
  • Joined: Feb 01, 2018
|
#60269
Hi AltheaD,
Answer C essentially voids the need for any of the premises in the stimulus. If C were correct then only the conclusion that was stated in the stimulus would matter. But we would also need to know that there was was a significantly increased demand for workers (which we don't).

Answer A is not contingent upon C, because nothing in the stimulus says that we need a significant increase in a demand for workers to significantly decrease unemployment. We just need the option for job opportunity to decrease unemployment.
Hope that helps!
-Malila
 altheaD
  • Posts: 17
  • Joined: Jun 08, 2018
|
#60377
Thank you! Basically I think I assumed that demand for workers is the ONLY way that unemployment can go down, but after hard thinking, sure it'd be possible that suddenly all the enemployed people went out and started their own independent business... then that's one possible scenario besides workers being in demand. I still think it's not an unreasonable assumption that workers don't get jobs unless there is a demand for them... after all, workers wouldn't get a job at a company where there isn't a demand at all for more... how is this not a commonsense sort of assumption that LSAC tells folks to work with...??
I'm finding that I tend to make similar erroneous assumptions...leading to wrong answers... Any tip would be appreciated!
 Malila Robinson
PowerScore Staff
  • PowerScore Staff
  • Posts: 296
  • Joined: Feb 01, 2018
|
#61756
Hi AltheaD,
From what you have just described you are thinking of assumptions as something sort of amorphous, as in, we don't know a lot about the thing being assumed, but it seems like it would flow from the conversation (which is the "normal" understanding of assumptions). But on the LSAT, a logical assumption is something that absolutely must be true in order for anything that has been stated in the stimulus to be true. It is essentially the base of an argument. So if you take it away, the argument falls into the abyss. ;)
As you pointed out there may be other ways to decrease unempolyment without relying on a demand for workers. Even if the possibility is a bit far fetched, it still means that an answer like C would not absolutely have to be true.

Hope that helps, if not, please reach out again!
-Malila
User avatar
 LawSchoolDream
  • Posts: 57
  • Joined: Jan 18, 2024
|
#105087
Alex Bodaken wrote: Thu Jun 07, 2018 4:11 pm EmiliaGrace,

Thanks for the question! Sure, we could diagram this one out. Here's what it would look like:

Premise 1: Government spending signifcantly increases :arrow: unemployment decreases
Premise 2: Government spending significantly decreases :arrow: unemployment decreases
Conclusion: Unemployment will soon decrease.

Note that the conclusion is the first sentence of the stimulus here - I know you said in your question you had a tough time picking it out.

Obviously this is a bit of a strange argument: the economist is basically saying that whether government spending significantly increases or significantly decreases, unemployment will decrease. So in order to prove the conclusion, we are looking for an assumption that one of the two premises will occur. And that is what answer choice (A) gives us: it says that either premise 1 or premise 2 will occur, and we know that if either of those occur, then unemployment will decrease, thereby validating the economist's conclusion.

Hope that's helpful!
Alex

Hi, I actually went with B. If this were an Assumption question, would B be correct? Also I Did extensive conditional reasoning like Gov spending decrease --> Economy stimulated short term --> Unemployment Down


How do I know how extensive I need to get with these conditionals?

And also I'm taking A LOT of time with many of these LR stimuluses. Many times I have to read a few times just to absorb but also have to write things down, its taking me too much time and tiring me out. Any tips on how to absorb and retain in the first go?
User avatar
 LawSchoolDream
  • Posts: 57
  • Joined: Jan 18, 2024
|
#105088
Administrator wrote: Fri Dec 09, 2016 10:47 pm Complete Question Explanation

Our author opens with his conclusion that unemployment will soon decrease. He then goes on to tell us that significantly increased government spending will lead to this result, and that significantly decreased government spending will also lead to that same result. These are his premises for the claim in the first sentence.

At first this Justify the Conclusion question comes off like a False Dilemma flaw question, and test takers may have braced themselves for that question stem and answer choice as soon as they noticed that the author made the classic mistake of ignoring the possibility that things could stay the same (or at least not change significantly). The government might spend something close to what they have been spending, neither increasing nor decreasing significantly.

In order to justify the conclusion, we must make this dilemma real - we have to eliminate the possibility of spending staying close to current amounts. If spending cannot stay the same or somewhat close to it, then it must go up or down significantly, either of which would lead to the conclusion. That must be our prephrase - spending cannot stay as it is but must change significantly one way or the other.

Answer A: This is the correct answer. A perfect match for our prephrase, eliminating the dilemma and proving that one way or another, unemployment will decrease.

Answer B: There is no need to assume anything about what policies are currently being implemented or contemplated or rejected. All we need to assume is that spending will change significantly in one direction or another.

Answer C: While this answer looks reasonable and makes perfect sense on its own, it doesn't tell us with any certainty that there will be such increased demand for workers, and so this does not prove that unemployment will decrease soon. It only shows us one way in which that might come about.

Answer D: This answer is outside the scope of the argument and does nothing to help justify our conclusion about what will happen soon. It gives additional information about what could happen later, but that's not what we are looking for.

Answer E: This answer looks like a Mistaken Negation of the two conditional claims in the argument, and as such it cannot justify our conclusion.
I chose B because I felt if policies are being implemented, then either government will increase spending or decrease it.

I skipped A because I felt A was more of a fact based Strongly supported answer.

Can you please clarify?
User avatar
 Jeff Wren
PowerScore Staff
  • PowerScore Staff
  • Posts: 389
  • Joined: Oct 19, 2022
|
#105334
Hi LawSchoolDream,

No Answer B would still be incorrect even for an Assumption question. The argument doesn't require anything about the government's intentions, which is what Answer B discusses. All that the argument requires is that one of these two sufficient conditions will happen, which is what Answer A states.

It may be helpful to look at a similar (but hopefully easier to follow) argument.

Premise: If it is raining, I will stay home.
Premise: If it is snowing, I will stay home.
Conclusion: Therefore, I will stay home.

What would we need to know in order to prove this conclusion (i.e. justify this argument)? We would need to know that it is either raining or snowing.

The argument in this question follows the exact same format.

As far as your diagramming, while you don't need to include the middle terms here, it's not really a problem to include them. The key is to realize that each of these conditionals leads to the same necessary (decreasing unemployment), which is the same as the conclusion that we are trying to prove.

You may be reading too fast or not carefully enough the first time. Also, while you generally shouldn't need to write anything down for most LR questions (except to diagram conditionals, such as this question), it can be helpful to digitally highlight or underline key words and the conclusion in the stimulus.

As for your reason for not choosing Answer A (that it is a fact based answer), this is not the correct way to approach Justify questions. Instead, you need to look at the argument that is given and determine what is missing from the argument to prove the conclusion. One tip is to try adding Answer A to the argument, and then reading the argument (moving the first sentence, which is the conclusion, to the very end for clarity).

Get the most out of your LSAT Prep Plus subscription.

Analyze and track your performance with our Testing and Analytics Package.